文档库 最新最全的文档下载
当前位置:文档库 › 大学物理习题集及解答(振动与波,波动光学)

大学物理习题集及解答(振动与波,波动光学)

大学物理习题集及解答(振动与波,波动光学)
大学物理习题集及解答(振动与波,波动光学)

1.有一弹簧,当其下端挂一质量为m的物体时,伸长量为9.8 10-2 m。若使物体上下振动,且规定向下为正方向。(1)t = 0时,物体在平衡位置上方8.0 10-2 m处,由静止开始向下运动,求运动方程。(2)t = 0时,物体在平衡位置并以0.60 m/s的速度向上运动,求运动方程。

题1分析:

求运动方程,也就是要确定振动

的三个特征物理量A、ω,和?。其中振动的角频率是由弹簧振子系统的固有性质(振子质量m及弹簧劲度系数k)决定的,即ω,k可根据物体受力平衡时弹簧的=

k/

m

伸长来计算;振幅A 和初相?需要根据初始

条件确定。

解:

物体受力平衡时,弹性力F 与重力P 的大

小相等,即F = mg 。 而此时

弹簧的伸长量m l 2108.9-?=?。 则

弹簧的劲度系数l mg l F k ?=?=//。

系统作简谐运动的角频率为

1s 10//-=?==l g m k ω

(1)设系统平衡时,物体所在处为坐标

原点,向下为x 轴正向。 由初始条件t = 0时,m x 210100.8-?=,010=v 可得振幅

m 100.8)/(2210102-?=+=ωv x A ;应用旋转矢量法可确定初相π?=1。则运动方程为

])s 10cos[()m 100.8(121π+?=--t x

(2)t = 0时,020=x ,

120s m 6.0-?=v ,同理可得m 100.6)/(22202022-?=+=ωv x A ,

2/2π?=;则运动方程为

]5.0)s 10cos[()m 100.6(122π+?=--t x

2.某振动质点的x -t 曲线如图所示,

试求:(1)运动方程;(2)点P 对应的相位;

(3)到达点P 相应位置所需要的时间。

题2分析:

由已知运动方程画振动曲线和由振动曲

线求运动方程是振动中常见的两类问题。

本题就是要通过x -t 图线确定振动的三个

特征量量A 、ω,和0?,从而写出运动方

程。 曲线最大幅值即为振幅A ;而ω、0?通

常可通过旋转矢量法或解析法解出,一般采用旋转矢量法比较方便 。

解:

(1)质点振动振幅A = 0.10 m 。 而由

振动曲线可画出t = 0和t = 4s 时旋转矢量,如图所示。 由图可见初相)或3/5(3/00π?π?=-=,而由()3

201ππω+=-t t

得1

s 24/5-=πω,则运动方程为 ??????-??? ??=-3s 245cos )m 10.0(1ππt x

(2)图(a )中点P 的位置是质点从A /2处运动到正向的端点处。 对应的旋转矢量

图如图所示。 当初相取3/0π?-=时,点 P

的相位为πω??2)0(p 0P =-+=t )。

(3)由旋转关量图可得3)0(P πω=-t ,

则s 6.1P =t

0)0(P 0P =-+=t ω??(如果初相取3/50π?=,则点P 相应的相位应表示为

π

ω??2)0(p 0P =-+=t

3. 点作同频率、同振幅的简谐运动。第一

个质点的运动方程为)cos(1

?ω+=t A x ,当第一个质点自振动正方向回到平衡位置时,第二个

质点恰在振动正方向的端点。试用旋转矢

量图表示它们,并求第二个质点的运动方

程及它们的相位差。

题3.解:图为两质点在特定

时刻t 的旋转矢量图,OM 表

示第一个质点振动的旋转矢

量;ON 表示第二个质点振动的旋转矢量。 可见第一个质点振动的相位比第二个质点

超前2/π,即它们的相位差2/π?=?。第二个质

点的运动方程应为

)2cos(2π?ω-+=t A x

4.波源作简谐运动,其运动方程为

t y )s 240cos()m 100.4(13--?=π,它所形成的波形以30 m/s

的速度沿一直线传播。(1)求波的周期及波

长;(2)写出波 动方程。

解:(1)由已知的运动方程可知,质点振

动的角频率1

s 240-=πω。根据分析中所述,波的周期就是振动的周期,故有

s 1033.8/23-?==ωπT

波长为

m 25.0==uT λ

(2) 将已知的波源运动方程与简谐运动

方程的一般形式比较后可得

0s 240m 100.4013==?=--?πω,,A

故以波源为原点,沿x 轴正向传播的波

的波动方程为

()[]

])m 8()s 240cos[()m 100.4(cos 1130x t u x t A y ----?=+-=ππ?ω

5.波源作简谐振动,周期为

s 100.12-?,以它经平衡位置向正方向运动时为时间起点,

若此振动以u = 400 m/s 的速度沿直线传播。求:(1)距离波源8.0 m 处质点P 的运动方程和初相;(2)距离波源9.0 m 和10.0 m 处两点的相位差。

解:在确

知角频率1s 200/2-==ππωT 、波速1s

m 400-?=u 和初相)或2/(2/30ππ?-=的条件下,波动方程

]2/3)s m 400/)(s 200cos[(11ππ+?-=--x t A y

位于 x P = 8.0 m 处,质点P 的运动方程为 ]

2/5)s 200cos[(1P ππ-=-t A y

该质点振动的初相2/50π?-=P 。而距波

源9.0 m 和 10.0 m 两点的相位差为

2//)(2/)(21212ππλπ?=-=-=?uT x x x x

如果波源初相取2/0π?-=,则波动方

程为

]

2/9)(s 200cos[(1ππ-=-t A y 质点P 振动的初相也变为2/9P0π?-=,但波线上任两点间的相位差并不改变。

6.平面简谐波以波速u = 0.5 m/s 沿Ox 轴负

方向传播,在t = 2 s 时

的波形图如图所示。求原点的运动方程。 题6分析:从波形图中可知振幅A 、波长λ和频率ν。由于图(a )是t = 2 s 时刻的波形曲线,因此确定t = 0时原点处质点的初

相就成为本题求解的难

点。求t = 0时的初相有多

种方法。下面介绍波形平

移法、波的传播可以形象

地描述为波形的传

播。由于波是沿 Ox

轴负向传播的,所以可将t = 2 s 时的波形沿Ox 轴正向平移m 0.1s 2)s m 50.0(1=??==?-uT x ,即得到t = 0时的波形图,再根据此时点O 的状态,用旋转关量法确定其初相位。

解:由图得知彼长m 0.2=λ,振幅A = 0.5

m 。角频率1

s 5.0/2-==πλπωu 。

按分析中所述,从图可知t = 0时,原点

处的质点位于平衡位置。并由旋转矢量图得到2/0π?=,则所求运动方程为

]5.0)s 5.0cos[()m 50.0(1ππ+=-t y

7. 牛顿环装置中,透镜的曲率半径R = 40 cm ,用单色光垂直照射,在

反射光中观察某一级暗环的半

径r = 2.5 mm 。现把平板玻璃

向下平移m 0.50μ=d ,上述被观察暗环的半径变为何值?

8. 在折射率52.13=n 的照相机镜头表面

涂有一层折射率38.12=n 的MgF 2增透

膜,若此膜仅适用于波长nm 550=λ的光,则此膜的最小厚度为多少?

解:(解法一)因干涉的互补性,波长为

550nm 的光在透射中得到加强,则在反射中一定减弱,两反射光的光程差d n 222=?,由干涉相消条件2)12(2λ+=?k ,得 24)12(n k d λ

+=

取k = 0,则nm 3.99min =d

(解法二)

由于空气的折射率n l = 1,且有n 1<n 2<n 3,则对透射光而言,两相干光的光程差2221λ

+=?d n ,由干涉加强条件,λk =?1得,

取k = l ,则膜的最小厚度nm 3.99min =d

9. 如图所示,狭缝的宽度 b =

0.60 mm ,透镜焦距f = 0.40 m ,

有一与狭缝平行的屏放置在透镜焦平面处。若以单色平行光垂直照射狭缝,则在屏上离点O 为x = 1.4 mm 处的点 P ,看到的是衍射明条纹。试求:(1)该入射光的波长;(2)点P 条纹的级数;(3)从点P 看来对该光波而言,狭缝处的波阵面可作半波带的数目。

题9分析:单缝衍射中的明纹条件为2)12(sin λ

?+±=k b ,在观察点P 确定(即?确

定)后,由于k 只能取整数值,故满足上式的λ只可取若干不连续的值,对照可见光的波长范围可确定入射光波长的取值。此外,如点P 处的明纹级次为k ,则狭缝处的波阵面可以划分的半波带数目为(2k + l ),它

们都与观察点P 有关,?越大,可以划分的

半波带数目也越大。

解:

(l )透镜到屏的距离为d ,由于d >>b ,对点P 而言,有d x ≈?sin 。根据单缝衍射明纹条件 2)12(sin λ

?+=k b ,有

2)12(λ+=k d bx 将b 、d (f d ≈)、x 的值代入,并考虑可见光波长的上、下限值,有

nm 400min =λ时,75.4max =k

nm 760max =λ时,27.2min =k

因k 只能取整数值,故在可见光范围内只允许有 k = 4和 k = 3,它们所对应的入射

光波长分别为2λ= 466.7 nm和1λ= 600 nm。

(2)点P的条纹级次随入射光波长而异,当1λ= 600 nm时,k = 3;当2λ= 466.7 nm时,k = 4。

(3)当1λ = 600 nm时,k = 3,半波带数目为(2k+l)= 7;当2λ= 466.7 nm时,k = 4,半波带数目为9。

10.为了测定一光栅的光栅常数,用λ= 632.8 nm的单色平行光垂直照射光栅,已知第一级明条纹出现在38的方向,试问此光栅的光栅常数为多少?第二级明条纹

出现在什么角度?若使用此光栅对某单色光进行同样的衍射实验,测得第一级明条纹出现在270的方向上,问此单色光的波长为多少?对此单色光,最多可看到第几级明条纹?

解:由题意知,在λ = 632.8 nm , k = 1

时,衍射角? = 380,由光栅方程可得光栅

常数 m 1003.1sin 6-?==?λk d

k = 2时,因12>d λ,第二级明纹(即k =

2)所对应的衍射角2?不存在,因此用

此波长的光照射光栅不会出现第二级明纹。 若用另一种波长的光照射此光栅,因第一

级明纹出现在0

27'=?的方向上,得 nm 468'sin '==k d ?λ

令1'sin =?,可得用此波长光照射时,屏上出现的最大条纹级次为

2.2'm ==λd

k

因k 只能取整数,则k m = 2,故最多只能看到第二级明纹。

11.测得一池静水的表面反射出来的太阳光是线偏振光,求此时太阳处在地平线的多大仰角处?(水的折射率为1.33)(36.9o)

12.一束光是自然光和线偏振光的混合,当它通过一偏振片时,发现透射光的强度取

决于偏振片的取向,其强度可以变化5倍,求入射光中两种光的强度各占总入射光强度的几分之几。

解:设入射混合光强为I ,其中线偏振光强为xI ,自然光强为(1

x )I 。按题意旋转偏振片,则有:

最大透射光强。

I x x I ??????+-=)1(21max 最小透射光强。 I x I ??????-=)1(21min

按题意,I max /I min = 5,则有 )1(215)1(21x x x -?=+-

解得 3/2=x

即线偏振光占总入射光强的2/3,自然光占1/3。

13.用波长为589.3 nm 的钠黄光观察牛顿环,测得某一明环的半径为1.0×10-3 m ,而其外第四个明环的半径为3.0×10-3 m ,求平凸透镜凸面的曲率半径。(3.39m )

14. 自然光射到平行平板玻璃上,反射光恰为线偏振光,且折射光的折射角为32o ,试求:

(1) 自然光的入射角;

(2) 玻璃的折射率;

(3) 玻璃下表面的反射光、透射光的偏振状态。

解 (1)由布儒斯特定律知,反射光为线偏振光时,反射光与折射光垂直,即o 0090=+r i

所以自然

光的入射角 o

0o 05890=-=r i

(2) 根据布儒斯特定律

120 tg n n i =,其中11=n ,所以玻璃折射率为6.158tg tg o 012===i n n

(3) 在玻璃片下表面,入射角等于

0r ,

折射角等于0i ,

因为 o 0090=+r i 120tg n n i = 所以

210tg n n r = 因此下表面的反射光也是线偏振光,振动方向垂直入射面,玻璃片的透射光还是部分偏振光,不过偏振度比在玻璃中更大了。

如图所示,在杨氏实验中,入射光的波长为

。今将折射率 的薄玻璃片覆盖在狭缝

上,这时观察到屏幕上零级明条纹向上移到原来的第7级明条纹处。

求此玻璃片厚度。

5大学物理习题_波动光学

波动光学 一、选择题 1.如图,折射率为2n ,厚度为e 的透明介质薄膜的上方和下方的透明介质的折射率分别为1n 和3n ,且3221,n n n n ><,1λ为入射光在1n 中的波长,当单色平行光1λ垂直入射到该薄膜上,则从薄膜上、下两表面反射的光束的光程差是: (A )e n 22; (B )11222n e n λ- ; (C )112212λn e n -; (D )122212λn e n - 2.单色平行光垂直照射在薄膜上,经上下两表面反射的两束光发生干涉,如图所示,若薄膜厚度为e ,且321n n n ><,1λ为入射光在1n 中的波长,则两束反射光在相遇点的位相差为 (A )1 122λπn e n ; (B )πλπ+1214n e n ; (C )πλπ+1124n e n ; (D )1124λπn e n 。 3.在双缝干涉实验中,入射光的波长为λ,用玻璃纸遮住双缝中的一个缝,若玻璃纸中光程比相同厚度的空气的光程大λ5.2,则屏上原来的明纹处 (A )仍为明条纹。 (B )变为暗条纹。 (C )既非明纹也非暗纹。 (D )无法确定是明纹,还是暗纹。 4.如图所示,用波长为λ的单色光照射双缝干涉实验装置,若将一折射率为n 、劈角为α的透明劈尖b 插入光线2中,则当劈尖b 缓慢地向上移动时(只遮住2S ),屏C 上的干涉条纹 (A )间隔变大,向下移动。 (B )间隔变小,向上移动。 (C )间隔不变,向下移动。 (D )间隔不变,向上移动。 λS 1S 2S O C 1 2 b 图 3 3

5.在杨氏双缝干涉实验中,如果在上方的缝后面贴一片薄的透明云母片,中央明纹会 (A )向上移动; (B )向下移动; (C )不移动; (D )向从中间向上、下两边移动。 6.白光垂直照射到空气中一厚度为nm 450的肥皂膜上。设肥皂的折射率为1.32,试问该膜的正面呈什么颜色: (A )紫光(nm 401)(B )红光(nm 668)(C )蓝光(nm 475)(D )黄光(nm 570) 7.如图示两个直径有微小差别的彼此平行的滚柱之间的距离为L ,夹在两块平晶的中间,形成空气劈尖,当单色光垂直入射时,产生等厚干涉条纹,如果滚柱之间的距离变小,则在L 范围内干涉条纹的 (A )数目减少,间距变大;(B )数目不变,间距变小; (C )数目增加,间距变小; (D )数目减少,间距不变。 8纹 (A )向劈尖平移,条纹间隔变小; (B )向劈尖平移, 条纹间隔不变; (C )反劈尖方向平移,条纹间隔变小;(D )反劈尖方向平移,条纹间隔不变。 9.波长为λ的平行单色光垂直照射到劈尖薄膜上,劈尖薄膜的折射率为n ,则第2级明纹与第5级明纹所对应的薄膜厚度之差为: (A )n 2λ ; (B )23λ; (C )n 23λ; (D )n 4λ。 10.根据惠更斯—菲涅耳原理,若已知光在某时刻的波阵面为S ,则S 的前方某点P 的光强度决定于波阵面S 上所有面积元发出的子波各自传到P 点的 (A )振动振幅之和; (B )光强之和; (C )振动振幅之和的平方; (D )振动的相干叠加。 11.在单缝夫琅和费衍射实验中,若减小缝宽,其他条件不变,则中央明条纹 (A )宽度变小; (B )宽度变大; (C )宽度不变,且中心强度也不变; (D )宽度不变,但中心强度变小。 12.用平行单色光垂直照射在单缝上时,可观察夫琅禾费衍射。若屏上点P 处为第二级明纹,则相应的单缝波阵面可分成的半波带数目为: (A )3个; (B )4个; (C )5个; (D )6个。

大学物理下册波动光学习题解答杨体强

波动光学习题解答 1-1 在氏实验装置中,两孔间的距离等于通过光孔的光波长的100倍,接收屏与 双孔屏相距50cm 。求第1 级和第3级亮纹在屏上的位置以及它们之间的距离。 解: 设两孔间距为d ,小孔至屏幕的距离为D ,光波波长为λ,则有=100d λ. (1)第1级和第3级亮条纹在屏上的位置分别为 -5150==510m 100D x d λ=?? -42503==1.510m 100 D x d λ=?? (2)两干涉条纹的间距为 -42=1.010m D x d λ?=?? 1-2 在氏双缝干涉实验中,用0 6328A =λ的氦氖激光束垂直照射两小孔,两小孔的间距为1.14mm ,小孔至屏幕的垂直距离为1.5m 。求在下列两种情况下屏幕上干涉条纹的间距。 (1)整个装置放在空气中; (2)整个装置放在n=1.33的水中。 解: 设两孔间距为d ,小孔至屏幕的距离为D ,装置所处介质的折射率为n ,则两小孔出射的光到屏幕的光程差为 21()x n r r nd D δ=-= 所以相邻干涉条纹的间距为 D x d n λ?=? (1)在空气中时,n =1。于是条纹间距为 943 1.5 632.8108.3210(m)1.1410 D x d λ---?==??=?? (2)在水中时,n =1.33。条纹间距为 9 43 1.563 2.810 6.2610(m)1.1410 1.33 D x d n λ---???=?==??? 1-3 如图所示,1S 、2S 是两个相干光源,它们到P 点的距离分别为1r 和2r 。路径1S P 垂直穿过一块厚度

为1t 、折射率为1n 的介质板,路径2S P 垂直穿过厚度为2t ,折射率为2n 的另一块介质板,其余部分可看做真空。这两条路径的光程差是多少? 解:光程差为 222111[r (n 1)t ][r (n 1)t ]+--+- 1-4 如图所示为一种利用干涉现象测定气体折射率的原理性结构,在1S 孔后面放 置一长度为l 的透明容器,当待测气体注入容器而将空气排出的过程中幕上的干涉条纹就会移动。由移过条纹的根数即可推知气体的折射率。 (1)设待测气体的折射率大于空气折射率,干涉条纹如何移动? (2)设 2.0l cm =,条纹移过20根,光波长为 589.3nm ,空气折射率为1.000276,求待测气体(氯气)的折射率。 1-5 用波长为500 nm 的单色光垂直照射到由两块光学平玻璃构成的空气劈尖上。在观察反射光的干涉现象中,距劈尖棱边1=1.56 cm 的A 处是从棱边算起的第四条暗条纹中心。 (1)求此空气劈尖的劈尖角θ; (2)改用600 nm 的单色光垂直照射到此劈尖上,仍观察反射光的干涉条纹,A 处是明条纹还是暗条纹? (3)在第(2)问的情形从棱边到A 处的围共有几条明纹,几条暗纹?

(完整版)《大学物理》习题册题目及答案第19单元波动光学

第19单元 波动光学(二) 学号 姓名 专业、班级 课程班序号 一 选择题 [C]1. 在如图所示的单缝夫琅和费衍射装置中,将单缝宽度a 稍稍变窄,同时使会聚透镜L 沿y 轴正方向作微小位移,则屏幕E 上的中央衍射条纹将 (A) 变宽,同时向上移动 (B) 变宽,同时向下移动 (C) 变宽,不移动 (D) 变窄,同时向上移动 (E) 变窄,不移动 [ D ]2. 在双缝衍射实验中,若保持双缝S1和S2的中心之间的距离d 不变,而把两条缝的宽度a 稍微加宽,则 (A) 单缝衍射的中央主极大变宽,其中所包含的干涉条纹数目变少 (B) 单缝衍射的中央主极大变宽,其中所包含的干涉条纹数目变多 (C) 单缝衍射的中央主极大变宽,其中所包含的干涉条纹数目不变 (D) 单缝衍射的中央主极大变窄,其中所包含的干涉条纹数目变少 (E) 单缝衍射的中央主极大变窄,其中所包含的干涉条纹数目变多 [ C ]3. 在如图所示的单缝夫琅和费衍射实验中,若将单缝沿透镜光轴方向向透镜平移,则屏幕上的衍射条纹 (A) 间距变大 (B) 间距变小 (C) 不发生变化 (D) 间距不变,但明暗条纹的位置交替变化 [ B ]4. 一衍射光柵对某一定波长的垂直入射光,在屏幕上只能出现零级和一级主极大,欲使屏幕上出现更高级次的主极大,应该 (A) 换一个光栅常数较小的光栅 (B) 换一个光栅常数较大的光栅 (C) 将光栅向靠近屏幕的方向移动 (D) 将光栅向远离屏幕的方向移动 λ L 屏幕 单缝 f 单缝 λa L E f O x y

[ B ]5. 波长λ =5500 ?的单色光垂直入射于光柵常数d = 2?10-4cm 的平面衍射光柵上,可能观察到的光谱线的最大级次为 (A) 2 (B) 3 (C) 4 (D) 5 二 填空题 1. 用半波带法讨论单缝衍射暗条纹中心的条件时,与中央明条纹旁第二个暗条纹中心相对应的半波带的数目是_____4_________。 2. 如图所示,在单缝夫琅和费衍射中波长λ的单色光垂 直入射在单缝上。若对应于汇聚在P 点的衍射光线在缝 宽a 处的波阵面恰好分成3个半波带,图中 ____________CD BC AB ==,则光线1和光线2在P 点的相差为 π 。 3. 一束单色光垂直入射在光栅上,衍射光谱中共出现5条明纹,若已知此光栅缝宽度与不透明部分宽度相等,那么在中央明纹一侧的两条明纹分别是第__一___级和第___三_级谱线。 4 用平行的白光垂直入射在平面透射光栅上时,波长为λ1=440nm 的第3级光谱线,将与波长为λ2 = 660 nm 的第2级光谱线重叠。 5. 用波长为λ的单色平行光垂直入射在一块多缝光柵上,其光柵常数d=3μm ,缝宽a =1μm ,则在单缝衍射的中央明条纹中共有 5 条谱线(主极大)。 三 计算题 1. 波长λ=600nm 的单色光垂直入射到一光柵上,测得第二级主极大的衍射角为30o ,且第三级是缺级。则 (1) 光栅常数(a +b )等于多少? (2) 透光缝可能的最小宽度a 等于多少 (3) 在选定了上述(a +b )和a 之后,求在屏幕上可能呈现的全部主极大的级次。 解:(1) 由光栅公式:λ?k d =sin ,由题意k = 2,得 P λ5.1λA B C D a 1234

《大学物理学》波动光学习题及答案

一、选择题(每题4分,共20分) 1.如图所示,波长为λ的平行单色光垂直入射在折射率为2n 的薄膜上,经上下两个表面反射的两束光发生干涉。若薄膜厚度为e ,而且321n n n >>,则两束反射光在相遇点的位相差为(B (A ) 22πn e λ ; (B ) 24πn e λ ; (C ) 24πn e πλ -; (D ) 24πn e πλ +。 2.如图示,用波长600λ=nm 的单色光做双缝实验,在屏P 处产生第五级明纹,现将折射率n =1.5的薄透明玻璃片盖在其中一条缝上,此时P (A )5.0×10-4cm ;(B )6.0×10-4cm ; (C )7.0×10-4cm ;(D )8.0×10-4cm 。 3.在单缝衍射实验中,缝宽a =0.2mm ,透镜焦距f =0.4m ,入射光波长λ=500nm 位置2mm 处是亮纹还是暗纹?从这个位置看上去可以把波阵面分为几个半波带?( D ) (A) 亮纹,3个半波带; (B) 亮纹,4个半波带;(C) 暗纹,3个半波带; (D) 暗纹,4个半波带。 4.波长为600nm 的单色光垂直入射到光栅常数为2.5×10-3mm 的光栅上,光栅的刻痕与缝宽相等,则光谱上呈现的全部级数为(B ) (A) 0、1±、2±、3±、4±; (B) 0、1±、3±;(C) 1±、3±; (D) 0、2±、4±。 5. 自然光以60°的入射角照射到某一透明介质表面时,反射光为线偏振光,则( B ) (A) 折射光为线偏振光,折射角为30°; (B) 折射光为部分偏振光,折射角为30°; (C) 折射光为线偏振光,折射角不能确定; (D) 折射光为部分偏振光,折射角不能确定。 二、填空题(每小题4分,共20分) 6.波长为λ的单色光垂直照射在空气劈尖上,劈尖的折射率为n ,劈尖角为θ,则第k 级明纹和第3k +级明纹的间距l = 32s i n λn θ 。 7.用550λ=nm 的单色光垂直照射牛顿环装置时,第4级暗纹对应的空气膜厚度为 1.1 μm 。 8.在单缝夫琅和费衍射实验中,设第一级暗纹的衍射角很小。若1600nm λ=为入射光,中央明纹宽度为 3m m ;若以2400nm λ=为入射光,则中央明纹宽度为 2 mm 。 9.设白天人的眼瞳直径为3mm ,入射光波长为550nm ,窗纱上两根细丝之间的距离为3mm ,人眼睛可以距离 13.4 m 时,恰能分辨。 10.费马原理指出,光总是沿着光程为 极值 的路径传播的。 三、计算题(共60分) 11.(10分)在杨氏双缝实验中,双缝间距d =0.20mm ,缝屏间距D =1.0m ,试求:(1)若第二级明条纹离屏中心的距离为6.0mm ,计算此单色光的波长;(2)相邻两明条纹间的距离. 解:(1)由λk d D x = 明知,23 0.26002110 x nm λ= =??, 3 n e

波动光学大学物理标准答案

习题13 13.1选择题 (1)在双缝干涉实验中,为使屏上的干涉条纹间距变大,可以采取的办法是[ ] (A) 使屏靠近双缝. (B) 使两缝的间距变小. (C) 把两个缝的宽度稍微调窄. (D) 改用波长较小的单色光源. [答案:C] (2)两块平玻璃构成空气劈形膜,左边为棱边,用单色平行光垂直入射.若上面的平玻璃以棱边为轴,沿逆时针方向作微小转动,则干涉条纹的[ ] (A) 间隔变小,并向棱边方向平移. (B) 间隔变大,并向远离棱边方向平移. (C) 间隔不变,向棱边方向平移. (D) 间隔变小,并向远离棱边方向平移. [答案:A] (3)一束波长为λ的单色光由空气垂直入射到折射率为n 的透明薄膜上,透明薄膜放在空气中,要使反射光得到干涉加强,则薄膜最小的厚度为[ ] (A) λ / 4 . (B) λ / (4n ). (C) λ / 2 . (D) λ / (2n ). [答案:B] (4)在迈克耳孙干涉仪的一条光路中,放入一折射率为n ,厚度为d 的透明薄片,放入后,这条光路的光程改变了[ ] (A) 2 ( n -1 ) d . (B) 2nd . (C) 2 ( n -1 ) d +λ / 2. (D) nd . (E) ( n -1 ) d . [答案:A] (5)在迈克耳孙干涉仪的一条光路中,放入一折射率为n 的透明介质薄膜后,测出两束光的光程差的改变量为一个波长λ,则薄膜的厚度是 [ ] (A) λ / 2 . (B) λ / (2n ). (C) λ / n . (D) λ / [2(n-1)]. [答案:D] 13.2 填空题 (1)如图所示,波长为λ的平行单色光斜入射到距离 为d 的双缝上,入射角为θ.在图中的屏中央O 处 (O S O S 21=),两束相干光的相位差为 ________________. [答案:2sin /d πθλ] (2)在双缝干涉实验中,所用单色光波长为λ=562.5 nm (1nm =10-9 m),双缝与观察屏的距离D =1.2 m ,若测得屏上相邻明条纹间距为?x =1.5 mm ,则双缝的间距d =

大学物理振动波动例题习题

精品 振动波动 一、例题 (一)振动 1.证明单摆是简谐振动,给出振动周期及圆频率。 2. 一质点沿x 轴作简谐运动,振幅为12cm ,周期为2s 。当t = 0时, 位移为6cm ,且向x 轴正方向运动。 求: (1) 振动表达式; (2) t = 0.5s 时,质点的位置、速度和加速度; (3)如果在某时刻质点位于x =-0.6cm ,且向x 轴负方向运动,求从该位置回到平衡位置所需要的时间。 3. 已知两同方向,同频率的简谐振动的方程分别为: x 1= 0.05cos (10 t + 0.75π) 20.06cos(100.25)(SI)x t π=+ 求:(1)合振动的初相及振幅. (2)若有另一同方向、同频率的简谐振动x 3 = 0.07cos (10 t +? 3 ), 则当? 3为多少时 x 1 + x 3 的振幅最大?又? 3为多少时 x 2 + x 3的振幅最小? (二)波动 1. 平面简谐波沿x 轴正方向传播,振幅为2 cm ,频率为 50 Hz ,波速为 200 m/s 。在t = 0时,x = 0处的质点正在平衡位置向y 轴正方向运动, 求:(1)波动方程 (2)x = 4 m 处媒质质点振动的表达式及该点在t = 2 s 时的振动速度。 2. 一平面简谐波以速度m/s 8.0=u 沿x 轴负方向传播。已知原点的振动曲线如图所示。求:(1)原点的振动表达式; (2)波动表达式; (3)同一时刻相距m 1的两点之间的位相差。 3. 两相干波源S 1和S 2的振动方程分别是1cos y A t ω=和2cos(/2)y A t ωπ=+。 S 1距P 点3个波长,S 2距P 点21/4个波长。求:两波在P 点引起的合振动振幅。

大学物理光学练习题及答案

光学练习题 一、 选择题 11. 如图所示,用厚度为d 、折射率分别为n 1和n 2 (n 1<n 2)的两片透明介质分别盖住杨氏双缝实验中的上下两缝, 若入射光的波长为, 此时屏上原来的中央明纹处被第三级明纹所占 据, 则该介质的厚度为 [ ] (A) λ3 (B) 1 23n n -λ (C) λ2 (D) 1 22n n -λ 17. 如图所示,在杨氏双缝实验中, 若用一片厚度为d 1的透光云母片将双缝装置中的上面一个缝挡住; 再用一片厚度为d 2的透光云母片将下面一个缝挡住, 两云母片的折射率均为n , d 1>d 2, 干涉条纹的变化情况是 [ ] (A) 条纹间距减小 (B) 条纹间距增大 (C) 整个条纹向上移动 (D) 整个条纹向下移动 18. 如图所示,在杨氏双缝实验中, 若用一片能透光的云母片将双缝装置中的上面一个缝盖住, 干涉条纹的变化情况是 [ ] (A) 条纹间距增大 (B) 整个干涉条纹将向上移动 (C) 条纹间距减小 (D) 整个干涉条纹将向 下移动 26. 如图(a)所示,一光学平板玻璃A 与待测工件B 之间形成空气劈尖,用波长λ=500nm(1nm = 10-9m)弯曲部分的顶点恰好与其右边条纹的直线部分的切线相切.则工件的上表面缺陷是 [ ] (A) 不平处为凸起纹,最大高度为500 nm (B) 不平处为凸起纹,最大高度为250 nm (C) 不平处为凹槽,最大深度为500 nm (D) 不平处为凹槽,最大深度为250 nm 43. 光波的衍射现象没有声波显著, 这是由于 [ ] (A) 光波是电磁波, 声波是机械波 (B) 光波传播速度比声波大 (C) 光是有颜色的 (D) 光的波长比声波小得多 53. 在图所示的单缝夫琅禾费衍射实验中,将单缝K 沿垂直光的入射光(x 轴)方向稍微 平移,则 [ ] (A) 衍射条纹移动,条纹宽度不变 (B) 衍射条纹移动,条纹宽度变动 (C) 衍射条纹中心不动,条纹变宽 (D) 衍射条纹不动,条纹宽度不变 K S 1 L L x a E f

大学物理题库-振动与波动

振动与波动题库 一、选择题(每题3分) 1、当质点以频率ν 作简谐振动时,它的动能的变化频率为( ) (A ) 2v (B )v (C )v 2 (D )v 4 2、一质点沿x 轴作简谐振动,振幅为cm 12,周期为s 2。当0=t 时, 位移为cm 6,且向x 轴正方向运动。则振动表达式为( ) (A) )(3 cos 12.0π π-=t x (B ) )(3 cos 12.0π π+=t x (C ) )(3 2cos 12.0π π-=t x (D ) ) (32cos 12.0π π+=t x 3、 有一弹簧振子,总能量为E ,如果简谐振动的振幅增加为原来的两倍,重物的质量增加为原来的四倍,则它的总能量变为 ( ) (A )2E (B )4E (C )E /2 (D )E /4 4、机械波的表达式为()()m π06.0π6cos 05.0x t y +=,则 ( ) (A) 波长为100 m (B) 波速为10 m·s-1 (C) 周期为1/3 s (D) 波沿x 轴正方向传播 5、两分振动方程分别为x 1=3cos (50πt+π/4) ㎝ 和x 2=4cos (50πt+3π/4)㎝,则它们的合振动的振幅为( ) (A) 1㎝ (B )3㎝ (C )5 ㎝ (D )7 ㎝ 6、一平面简谐波,波速为μ=5 cm/s ,设t= 3 s 时刻的波形如图所示,则x=0处的质点的振动方程为 ( ) (A) y=2×10- 2cos (πt/2-π/2) (m) (B) y=2×10- 2cos (πt + π) (m) (C) y=2×10- 2cos(πt/2+π/2) (m) (D) y=2×10- 2cos (πt -3π/2) (m) 7、一平面简谐波,沿X 轴负方向 传播。x=0处的质点 的振动曲线如图所示,若波函数用余弦函数表示,则该波的初位相为( ) (A )0 (B )π (C) π /2 (D) - π /2 8、有一单摆,摆长m 0.1=l ,小球质量g 100=m 。设小球的运动可看作筒谐振动,则该振动的周期为( ) (A) 2π (B )32π (C )102π (D )52π 9、一弹簧振子在光滑的水平面上做简谐振动时,弹性力在半个周期内所做的功为 [ ] (A) kA 2 (B )kA 2 /2 (C )kA 2 /4 (D )0

大学物理振动与波动

振动与波动 选择题 0580.一长为l 的均匀细棒悬于通过其一端的光滑水平固定轴上,(如图所示), 作成一复摆.已知细棒绕通过其一端的轴的转动惯量23 1 ml J =,此摆作微小振 动的周期为 (A) g l π2. (B) g l 22π. (C) g l 322π . (D) g l 3π. [ C ] 3001. 把单摆摆球从平衡位置向位移正方向拉开,使摆线与竖直方向成一微小角度θ ,然后由静止放手任其振动,从放手时开始计时.若用余弦函数表示其运动方程,则该单摆振动的初相为 (A) π. (B) π/2. (C) 0 . (D) θ. [ C ] 3003.轻弹簧上端固定,下系一质量为m 1的物体,稳定后在m 1下边又系一质量为m 2 的物体,于是弹簧又伸长了?x .若将m 2移去,并令其振动,则振动周期为 (A) g m x m T 122?π= . (B) g m x m T 212?π=. (C) g m x m T 2121?π= . (D) g m m x m T )(2212+π=?. [ B ] 3004.劲度系数分别为k 1和k 2的两个轻弹簧串联在一起,下面挂着质量为m 的物体,构成一个竖挂的弹簧振子,则该系统的振动周期为 (A) 21212)(2k k k k m T +π =. (B) ) (221k k m T +π= . (C) 2121)(2k k k k m T +π=. (D) 2 122k k m T +π=. [ C ] 3255.如图所示,在一竖直悬挂的弹簧下系一质量为m 的物体,再用此弹簧改系一质量为4m 的物体,最后将此弹簧截断为两个等长的弹簧并联后悬挂质 量为m 的物体,则这三个系统的周期值之比为 (A) 1∶2∶2/1. (B) 1∶2 1 ∶2 .

(完整版)大学物理波动光学的题目库及答案

一、选择题:(每题3分) 1、在真空中波长为λ的单色光,在折射率为n 的透明介质中从A 沿某路径传播到B ,若 A 、 B 两点相位差为3π,则此路径AB 的光程为 (A) 1.5 λ. (B) 1.5 λ/ n . (C) 1.5 n λ. (D) 3 λ. [ ] 2、在相同的时间内,一束波长为λ的单色光在空气中和在玻璃中 (A) 传播的路程相等,走过的光程相等. (B) 传播的路程相等,走过的光程不相等. (C) 传播的路程不相等,走过的光程相等. (D) 传播的路程不相等,走过的光程不相等. [ ] 3、如图,S 1、S 2是两个相干光源,它们到P 点的距离分 别为r 1和r 2.路径S 1P 垂直穿过一块厚度为t 1,折射率为n 1 的介质板,路径S 2P 垂直穿过厚度为t 2,折射率为n 2的另一 介质板,其余部分可看作真空,这两条路径的光程差等于 (A) )()(111222t n r t n r +-+ (B) ])1([])1([211222t n r t n r -+--+ (C) )()(111222t n r t n r --- (D) 1122t n t n - [ ] 4、真空中波长为λ的单色光,在折射率为n 的均匀透明媒质中,从A 点沿某一路径 传播到B 点,路径的长度为l .A 、B 两点光振动相位差记为?φ,则 (A) l =3 λ / 2,?φ=3π. (B) l =3 λ / (2n ),?φ=3n π. (C) l =3 λ / (2n ),?φ=3π. (D) l =3n λ / 2,?φ=3n π. [ ] 5、如图所示,波长为λ的平行单色光垂直入射在折射率为n 2的薄膜上,经上下两个表面反射的两束光发生干涉.若薄膜厚度为e ,而且n 1>n 2>n 3,则两束反射光在相遇点的相位差为 (A) 4πn 2 e / λ. (B) 2πn 2 e / λ. (C) (4πn 2 e / λ) +π. (D) (2πn 2 e / λ) -π. [ ] 6、如图所示,折射率为n 2、厚度为e 的透明介质薄膜的上方和下方的透明介质的折射率分别为n 1和n 3,已知n 1 <n 2<n 3.若用波长为λ的单色平行光垂直入射到该薄膜上,则从薄膜上、下两表面反射的光束①与②的光程差是 (A) 2n 2 e . (B) 2n 2 e -λ / 2 . (C) 2n 2 e -λ. (D) 2n 2 e -λ / (2n 2). [ ] 7、如图所示,折射率为n 2、厚度为e 的透明介质薄膜的上方和下方的透明介质的折射率分别为n 1和n 3,已知n 1< n 2> n 3.若用波长为λ的单色平行光垂直入射到该薄膜上,则从薄膜上、下两表面反射的光束(用①与②示意)的光程差是 (A) 2n 2 e . (B) 2n 2 e -λ / 2. (C) 2n 2 e -λ . (D) 2n 2 e -λ / (2n 2). P S 1S 2 r 1 n 1 n 2 t 2 r 2 t 1 n 1 3λ n 3 n 3

大学物理习题解答8第八章振动与波动 (1)

第八章 振动与波动 本章提要 1. 简谐振动 · 物体在一定位置附近所作的周期性往复运动称为机械振动。 · 简谐振动运动方程 ()cos x A t ω?=+ 其中A 为振幅,ω 为角频率,(ωt+?)称为谐振动的相位,t =0时的相位? 称为初相位。 · 简谐振动速度方程 d () d sin x v A t t ωω?= =-+ · 简谐振动加速度方程 2 2 2d ()d cos x a A t t ωω?= =-+ · 简谐振动可用旋转矢量法表示。 2. 简谐振动的能量 · 若弹簧振子劲度系数为k ,振动物体质量为m ,在某一时刻m 的位移为x ,振动速度为v ,则振动物体m 动能为 2 12k E m v = · 弹簧的势能为 2 12p E kx = · 振子总能量为 P 2 2 2 2 2 211()+()22 1=2 sin cos k E E E m A t kA t kA ωω?ω?=+=++ 3. 阻尼振动

· 如果一个振动质点,除了受弹性力之外,还受到一个与速度成正比的阻尼作用,那么它将作振幅逐渐衰减的振动,也就是阻尼振动。 · 阻尼振动的动力学方程为 2 2 2d d 20d d x x x t t β ω++= 其中,γ是阻尼系数,2m γ β= 。 (1) 当22ωβ>时,振子的运动一个振幅随时间衰减的振动,称阻尼振动。 (2) 当22ωβ=时,不再出现振荡,称临界阻尼。 (3) 当22ωβ<时,不出现振荡,称过阻尼。 4. 受迫振动 · 振子在周期性外力作用下发生的振动叫受迫振动,周期性外力称驱动力 · 受迫振动的运动方程为 2 2 P 2d d 2d d cos x x F x t t t m β ωω++= 其中,2k m ω=,为振动系统的固有频率;2C m β=;F 为驱动力振幅。 · 当驱动力振动的频率p ω等于ω时,振幅出现最大值,称为共振。 5. 简谐振动的合成与分解 (1) 一维同频率的简谐振动的合成 若任一时刻t 两个振动的位移分别为 111()cos x A t ω?=+ 222()cos x A t ω?=+ 合振动方程可表示为 ()cos x A t ω?=+ 其中,A 和? 分别为合振动的振幅与初相位 A =

西北工业大学大学物理作业答案6波动光学10

第六次作业 波动光学 一、选择题: 1.C ;2.A ;3.C ;4. BC ;5. A ;6. E ;7. C ;8. C ;9. A 。 二、填空题: 1. nr , 光程。 2. )(12r r n - , c r r n ν π )(212- 。 3. 频率相同、振动方向相同、相位差恒定的两束光;将同一光源发出的光分为两束,使两束光在空间经不同路程再次相遇;分波阵面;分振幅。 4. 5 5.1 。 5. 暗, 明,2 2n λ , sin θ 2θ 222n n λ λ 或 。 6. 光疏,光密,反射,或半波长2 λ ,π 。 7. 6,1 ,明。 8. 2, 4 1,?45。 9. 51370', 90o ,1.32 。 10. 610371.1-?m 。 11. 910699-?.m 。 12. 寻常;非常;光轴;O 。 三、问答题 答:将待检光线垂直入射偏振片,并以入射光为轴旋转偏振片,透射光强若光强不变则为自然光,光强有强弱变化但最弱不为零则为部分偏振光,光强有强弱变化且最弱处光强为零则为完全偏光。 四、计算题 1. 解:方法一:设相邻两条明纹间距为l ,则 10 b l = ,且L d = ≈θθtan sin 对于空气劈尖,相邻两条明纹对应的厚度差为 2 λ =?e 而 10 22sin b d L e l = = = ?=λθ λ θ 所以,细丝直径 m b L d 6 3 9 2 10 91710 008010 863210002055----?=?????= = ....λ

方法二: 由明纹条件得 λ λ δk e =+ =2 2 22??? ? ? -=λλk e k θλλθ22??? ? ? -== k e l k k 22)10(10??? ? ? -+=+λλk e k θ λλθ 22)10(10 10??? ? ? -+== ++k e l k k d L L d l l b k k λλθ λ5/521010= == -=+ 所以,细丝直径 m b L d 6 3 9 2 10 91710 008010 863210002055----?=?????= = ....λ 2. 解:(1)光程差2 21λ δ+ =e n ; 明纹条件 ) ,3,2,1(2 22 21 ==+ =k k e n λ λ δ 将最高点h e =代入得: 352 1 5768646122 121..=+??= += λ h n k 即:最高点为不明不暗,边缘处为暗环。 共有k =1、2、3、4、5 的5条明纹(干涉图样为同心圆环) 对应于k 的油膜厚度e k 为: nm k k n e k )2 1(180)2 1(21 - ?=- = λ k =1, e 1 = 90nm ; k =2, e 2 = 270nm ; k =3, e 3 = 450nm ; k =4, e 4 = 630nm ; k =5, e 5 = 810nm 。 (2) h = 864nm ,k = 5.3为非整数,条纹介于明暗之间,非明非暗条纹; h = 810nm ,2 10 52880nm 25768106.122 21λ λλ δ===+ ??=+=e n ,k = 5,为明纹; h = 720nm ,2 9 54nm 59222 5767206122 21λ λλ δ===+??=+ =..e n ,k = 4,为暗纹; 故最高点条纹变化为: 明暗之间→明纹→暗纹

大学物理复习题答案(振动与波动)

大学物理1复习题答案 一、单选题(在本题的每一小题备选答案中,只有一个答案是正确的,请把你认为正确答案的题号,填入题干的括号内) 1.一个弹簧振子和一个单摆(只考虑小幅度摆动),在地面上的固有振动周期分别为T 1和 T 2。将它们拿到月球上去,相应的周期分别为'T 1和'T 2。则有 ( B ) A .'T T >11且 'T T >22 B .'T T =11且 'T T >22 C .'T T <11且 'T T <22 D .'T T =11且 'T T =22 2.一物体作简谐振动,振动方程为cos 4x A t ?? =+ ?? ? πω,在4 T t = (T 为周期)时刻,物体的加速度为 ( B ) A. 2ω 2ω C. 2ω 2ω 3.一质点作简谐振动,振幅为A ,在起始时刻质点的位移为/2A -,且向x 轴的正方向 运动,代表此简谐振动的旋转矢量图为 ( D ) A A A A A A C) A x x A A x A B C D 4. 两个质点各自作简谐振动,它们的振幅相同、周期相同.第一个质点的振动方程为 )cos(1αω+=t A x .当第一个质点从相对于其平衡位置的正位移处回到平衡位置时,第二 个质点正在最大正位移处.则第二个质点的振动方程为 ( B ) A. )π21cos( 2++=αωt A x B. )π21 cos(2-+=αωt A x . C. )π2 3 cos( 2-+=αωt A x D. )cos(2π++=αωt A x .

5.波源作简谐运动,其运动方程为t y π240cos 10 0.43 -?=,式中y 的单位为m ,t 的单 位为s ,它所形成的波形以s m /30的速度沿一直线传播,则该波的波长为 ( A ) A .m 25.0 B .m 60.0 C .m 50.0 D .m 32.0 6.已知某简谐振动的振动曲线如图所示,位移的单位为厘米,时间单位为秒。则此简谐振动的振动方程为: ( B ) A .cos x t ππ??=+ ???2 2233 B .cos x t ππ??=+ ??? 42233 C .cos x t ππ??=- ???22233 D .cos x t ππ??=- ??? 42233 二. 填空题(每空2分) 1. 简谐运动方程为)4 20cos(1.0π π+ =t y (t 以s 计,y 以m 计) ,则其振幅为 0.1 m,周期为 0.1 s ;当t=2s 时位移的大小为205.0m. 2.一简谐振动的旋转矢量图如图所示,振幅矢量长2cm ,则该简谐振动 的初相为4 0π ?=,振动方程为_)4 cos(2π π+ =t y 。 3. 平面简谐波的波动方程为()x t y ππ24cos 08.0-=,式中y 和x 的单位为m ,t 的单位为s ,则该波的振幅A= 0.08 ,波长=λ 1 ,离波源0.80m 及0.30m 两处的相位差=?? -Л 。 4. 一简谐振动曲线如图所示,则由图可确定在t = 2s 时刻质点的位移为___0 ___,速度为:πω3=A . t

大学物理光学练习题及答案

大学物理光学练习题及 答案 -CAL-FENGHAI-(2020YEAR-YICAI)_JINGBIAN

光学练习题 一、 选择题 11. 如图所示,用厚度为d 、折射率分别为n 1和n 2 (n 1<n 2)的两片透明介质分别盖住杨氏双缝实验中的上下两缝, 若入射光的波长为, 此时屏上原来的中央明纹处被第三级明纹所占据, 则该介质的厚度为 [ ] (A) λ3 (B) 1 23n n -λ (C) λ2 (D) 1 22n n -λ 17. 如图所示,在杨氏双缝实验中, 若用一片厚度为d 1 的透光云母片将双缝装置中的上面一个缝挡住; 再用一 片厚度为d 2的透光云母片将下面一个缝挡住, 两云母片 的折射率均为n , d 1>d 2, 干涉条纹的变化情况是 [ ] (A) 条纹间距减小 (B) 条纹间距 增大 (C) 整个条纹向上移动 (D) 整个条纹向下移动 18. 如图所示,在杨氏双缝实验中, 若用一片能透光的 云母片将双缝装置中的上面一个缝盖住, 干涉条纹的变 化情况是 [ ] (A) 条纹间距增大 (B) 整个干涉条纹将 向上移动 (C) 条纹间距减小 (D) 整个干涉条纹将向下移动 26. 如图(a)所示,一光学平板玻璃A 与待测工件B 之间形成空气劈尖,用波长λ=500nm(1nm = 10-9m)(b)所示.有些条纹弯曲部分的顶点恰好与其右边条纹的直线部分的切线相切.则工件的上表面缺 陷是 [ ] (A) 不平处为凸起纹,最大高度为500 nm (B) 不平处为凸起纹,最大高度为250 nm (C) 不平处为凹槽,最大深度为500 nm (D) 不平处为凹槽,最大深度为250 nm 43. 光波的衍射现象没有声波显著, 这是由于 [ ] (A) 光波是电磁波, 声波是机械波 (B) 光波传播速度比声波大 (C) 光是有颜色的 (D) 光的波长比声波小得多 53. 在图所示的单缝夫琅禾费衍射实验中,将单缝K 沿垂直光的入射光(x )方向稍微平移,则 x a E

大学物理答案波动光学一

第十二章(一) 波动光学 一、选择题 1.C 2.A 3.C 4.E 5.D 6.D 7.B 8.B 二、填空题 1.1 mm 2.频率相同; 振动方向相同; 相位相等或相位差恒定; 相干光在相遇点的相位差等于π的偶数倍; 相干光在相遇点的相位差等于π的奇数倍。 3.向棱边移动; 向远离棱边移动; 向棱边移动且条纹间距减小,条纹变密。 4.71022.1-? m 5.λ d 2 6.6; 暗; a f λ3± 7.单缝处波前被分成的波带数越多,每个波带面积越小。 8.3 mm 三、计算题 1.解: 由 λλ k e n =+222 得 1 242-=k e n λ 由此可分别求得相应于k =1,2,3,4的波长为: 22401=λnm ; 7.7462=λnm ; 4483=λnm ; 3204=λnm 、 2λ3λ在可见光范围(400nm-760nm )内,故波长为746.7nm 和448nm 的两种光在反射时加强。 2.解:(1)m 11.010 2105502102249 10=?????==?∴=--x x d kD x k λ (2)0)(12=-+-e ne r r ()m 10828.3158.1106.6)1(6612--?=-??=-=-n e r r 71055010828.39 612≈??=-= ∴--λr r k 3.解: 2)12(2220λ λ +=++k e e 由几何关系R r e 22 = 代入,得:R e k r )2(0-= λ 其中,k 为整数,且λ02e k >

4.解: ()212s i n λ θ+k a ±= 2,1=k 得 1 2100.3m 4.01020.112105.0212212sin 26 33+?=??+??=+≈+=---k k f x k a k a ?λm 令k =1 10001=λnm (红外光) 令k =2 6002=λnm (黄光) 令k =3 6.4283=λnm (紫光) 题给入射光是紫色平行光,所以观察到的波长为428.6nm 即为第三级明条纹。又因k =3,则 ()2 7212sin λλθ=+k a = 所以,对应于这个衍射方向,可以把单缝处的波前分为7个波带。

《大学物理》习题册题目及答案第单元波动光学副本

第18单元 波动光学(一) 学号 姓名 专业、班级 课程班序号 一 选择题 [ A ]1. 如图所示,折射率为2n 、厚度为e 的透明介质薄膜的上方和下方的透明介质折射率分别为1n 和3n ,已知321n n n <<。若用波长为λ的单色平行光垂直入射到该薄膜上,则从薄膜上、下两表面反射的光束①与②的光程差是 (A) 22n e (B) 2e n 2λ- 21 (C) 22n e λ- (D) 22n e 2 2n λ - [ A ]2. 双缝干涉的实验中,两缝间距为d ,双缝与屏幕之间的距离为D (D >>d ),单色光波长为λ,屏幕上相邻的明条纹之间的距离为 (A) d D λ (B) D d λ (C) d D 2λ (D) D d 2λ [ B ]3. 如图,1S 、2S 是两个相干光源,它们到P 点的距离分别为 1r 和2r 。路径1S P 垂直穿过一块厚度为1t 、折射率为1n 的介质板,路径P S 2垂直穿过厚度为2t 、折射率为2n 的另一块介质板,其余部分可看作真空,这两条路径的光程差等于 (A) )()(111222t n r t n r +-+ (B) ])1([])1([111222t n r t n r -+--+ (C) )()(111222t n r t n r --- (D) 1122t n t n - [ C ]4. 如图所示,平行单色光垂直照射到薄膜上,经上下两表面反射的两束光发生干涉,若薄膜的厚度为e ,并且321n n n ><, 1λ 为入射光在折射率为n 1的媒质中的波长,则两束反射光在相遇点的相位差为 (A) 1122λπ n e n (B) πλπ+1212n e n (C) πλπ+1124n e n (D) 1 124λπn e n 。 [ B ]5. 如图,用单色光垂直照射在观察牛顿环的装置上。当平凸透镜垂直向上缓慢平移而远离平面玻璃时,可以观察到这些环状干涉条纹 (A) 向右平移 (B) 向中心收缩 (C) 向外扩张 (D) 静止不动 (E) 向左平移 [ D ]6. 在迈克尔逊干涉仪的一支光路中,放入一片折射率为n 的透明介质薄膜后,测出两束光的光程差的改变量为一个波长?,则薄膜的厚度是 (A) 2λ (B) n 2λ (C) n λ (D) )1(2-n λ 二 填空题 1 λe 1 n 2n 3 单色光 O . λ e 1 n 2n 3 ① ② S 1 S 2 1r 2 r 1n 2n 1 t 2 t P

精选-大学物理振动与波练习题与答案

第二章 振动与波习题答案 12、一放置在水平桌面上的弹簧振子,振幅2 10 0.2-?=A 米,周期50.0=T 秒,当0 =t 时 (1) 物体在正方向的端点; (2) 物体在负方向的端点; (3) 物体在平衡位置,向负方向运动; (4) 物体在平衡位置,向正方向运动。 求以上各种情况的谐振动方程。 【解】:π=π = ω45 .02 )m () t 4cos(02.0x ?+π=, )s /m ()2 t 4cos(08.0v π+?+ππ= (1) 01)cos(=?=?,, )m () t 4cos(02.0x π= (2) π=?-=?,1)cos(, )m () t 4cos(02.0x π+π= (3) 2 1)2cos(π=?-=π+?, , )m () 2 t 4cos(02.0x π+π= (4) 21)2cos(π-=?=π+?, , )m () 2 t 4cos(02.0x π-π= 13、已知一个谐振动的振幅02.0=A 米,园频率πω 4=弧度/秒, 初相2/π=?。 (1) 写出谐振动方程; (2) 以位移为纵坐标,时间为横坐标,画出谐振动曲线。 【解】:)m () 2 t 4cos(02.0x π+π= , )(2 12T 秒=ωπ= 15、图中两条曲线表示两个谐振动 (1) 它们哪些物理量相同,哪些物理量不同? (2) 写出它们的振动方程。

【解】:振幅相同,频率和初相不同。 虚线: )2 t 2 1cos(03.0x 1π-π= 米 实线: t cos 03.0x 2π= 米 16、一个质点同时参与两个同方向、同频率的谐振动,它们的振动方程为 t 3cos 4x 1= 厘米 )3 2t 3cos(2x 2π+= 厘米 试用旋转矢量法求出合振动方程。 【解】:)cm () 6 t 3cos(32x π+= 17、设某一时刻的横波波形曲线如图所示,波动以1米/秒的速度沿水平箭头方向传播。 (1) 试分别用箭头表明图中A 、B 、C 、D 、E 、F 、H 各质点在该时刻的运动方向; (2) 画出经过1秒后的波形曲线。 【解】: 18、波源作谐振动,其振动方程为(m ))240(1043t cos y π-?=,它所形成的波以30m/s 的速度沿一直线传播。

相关文档
相关文档 最新文档